Difference between revisions of "2023 AMC 10B Problems/Problem 6"

(Solution 1)
(Problem)
Line 6: Line 6:
  
  
On my copy of the AMC 10B, the order of the answers is different, so the correct answer is (E) 674  
+
On my copy of the AMC 10B, the order of the answers is different, so the correct answer is (E) 674
 +
 
 +
==Solution 1==
 +
 
 +
We calculate more terms:
 +
 
 +
<math>1,3,4,5,9,14,...</math>
 +
 
 +
We find a pattern: if <math>n</math> is a multiple of <math>3</math>, then the term is even, or else it is odd.
 +
There are <math>\lfloor \frac{2023}{3} \rfloor =\boxed{\textbf{(B) }674}</math> multiples of <math>3</math> from <math>1</math> to <math>2023</math>.
 +
 
 +
~Mintylemon66
  
 
==Solution 2==
 
==Solution 2==

Revision as of 19:50, 15 November 2023

Problem

Let $L_{1}=1, L_{2}=3$, and $L_{n+2}=L_{n+1}+L_{n}$ for $n\geq 1$. How many terms in the sequence $L_{1}, L_{2}, L_{3},...,L_{2023}$ are even?

$\textbf{(A) }673\qquad\textbf{(B)} 674\qquad\textbf{(C) }675\qquad\textbf{(D) }1010\qquad\textbf{(E) }1011$


On my copy of the AMC 10B, the order of the answers is different, so the correct answer is (E) 674

Solution 1

We calculate more terms:

$1,3,4,5,9,14,...$

We find a pattern: if $n$ is a multiple of $3$, then the term is even, or else it is odd. There are $\lfloor \frac{2023}{3} \rfloor =\boxed{\textbf{(B) }674}$ multiples of $3$ from $1$ to $2023$.

~Mintylemon66

Solution 2

Like in the other solution, we find a pattern, except in a more rigorous way. Since we start with $1$ and $3$, the next term is $4$.

We start with odd, then odd, then (the sum of odd and odd) even, (the sum of odd and even) odd, and so on. Basically the pattern goes: odd, odd, even, odd odd, even, odd, odd even…

When we take $\frac{2023}{3}$ we get $674$ with a remainder of one. So we have $674$ full cycles, and an extra odd at the end.

Therefore, there are $\boxed{\textbf{(B) }674}$ evens.

~e_is_2.71828